Ordinary Differential equations question

Click For Summary
The discussion centers on a third-order constant coefficient homogeneous differential equation with solutions y1, y2, and y3. It is established that the Wronskian W(y1, y2)(t) is positive for all real t, while W(y1, y2, y3)(0) equals zero. This indicates a linear dependence between the solutions, suggesting that y3 can be expressed as a linear combination of y1 and y2. Participants reference Abel's theorem to understand the implications of the Wronskian values. The inquiry highlights the importance of evaluating the Wronskian at specific points to determine solution relationships.
Hutchyy
Messages
4
Reaction score
0
#17 If you can't see the picture: Suppose that y1, y2, and y3 are solutions to a third order constant coefficient homogeneous differential equation. Suppose further that for all real t, W(y1,y2)(t)>0, but also W(y1,y2,y3)(0)=0. Then there exists c1 and c2 such that c1y1(t) + c2y2(t) =y3(t) for all real t. Is this true, false or maybe true? I know it must have something to do with abel’s theorem but I can’t really figure out how it applies.. ,

what is the significance of plugging in 0 for ’t’ in the wronskian?

I'm thinking its something that has to do with abel's theorem but I can't make any connections as to how having a positive wronskian relates to a bigger wronskian with 0 plugged into equalling 0. https://www.physicsforums.com/attachments/90663
 
Physics news on Phys.org
There's something wrong with your attachment. I can't view it.
 
Screen Shot 2015-10-23 at 11.37.35 PM.png
Screen Shot 2015-10-23 at 11.37.35 PM.png
 
When y= 0 (along the x-axis), dy/dx= 0. Which of the direction fields has that property? When x= 0 (along the y axis) dy/dx= y^2. Which of the direction fields has that property?
 
Thanks halls of ivy! But I got that one! I meant number 17 the word problem
 
Question: A clock's minute hand has length 4 and its hour hand has length 3. What is the distance between the tips at the moment when it is increasing most rapidly?(Putnam Exam Question) Answer: Making assumption that both the hands moves at constant angular velocities, the answer is ## \sqrt{7} .## But don't you think this assumption is somewhat doubtful and wrong?

Similar threads

Replies
19
Views
4K
Replies
1
Views
2K
  • · Replies 7 ·
Replies
7
Views
2K
  • · Replies 4 ·
Replies
4
Views
2K
  • · Replies 1 ·
Replies
1
Views
2K
  • · Replies 2 ·
Replies
2
Views
1K
  • · Replies 2 ·
Replies
2
Views
2K
  • · Replies 1 ·
Replies
1
Views
1K
  • · Replies 2 ·
Replies
2
Views
3K
  • · Replies 1 ·
Replies
1
Views
2K